0 Daumen
2k Aufrufe

Die Reihe \( \sum \limits_{n=0}^{\infty} a_{n} \) sei konvergent und die Folge \( \left(b_{n}\right)_{n \in \mathbb{N}} \) sei beschränkt. Ist dann die Reihe \( \sum \limits_{n=0}^{\infty} a_{n} b_{n} \) konvergent? Begründen Sie Ihre Aussage.

Avatar von

1 Antwort

+1 Daumen
 
Beste Antwort

$$ \text{Wähle }a_n=b_n=\frac{(-1)^n}{\sqrt{n+1}} $$

Die Reihe ist:

$$ \sum_{n=0}^\infty a_nb_n=\sum_{n=0}^\infty\frac1{n+1} $$

Die Reihe ist nicht konvergent.

Avatar von

Ein anderes Problem?

Stell deine Frage

Willkommen bei der Mathelounge! Stell deine Frage einfach und kostenlos

x
Made by a lovely community